AAFP Missed Questions

अब Quizwiz के साथ अपने होमवर्क और परीक्षाओं को एस करें!

A 64-year-old African-American male presents with persistent pleuritic pain. The patient does not feel well in general and has had a low-grade fever of around 100°F (38°C). His medications include simvastatin (Zocor), lisinopril (Prinivil, Zestril), low-dose aspirin, spironolactone (Aldactone), furosemide (Lasix), isosorbide mononitrate (Imdur), hydralazine, carvedilol (Coreg), and nitroglycerin as needed. A chest radiograph is normal and does not demonstrate a pneumothorax. Further evaluation rules out pulmonary embolus, pneumonia, and myocardial infarction. A diagnosis of pleurisy is made. Which one of the patient's medications could be related to this condition? A. Hydralazine B. Simvastatin C. Lisinopril D. Spironolactone E. Carvedilol

*A. Hydralazine* Several drugs are associated with drug-induced pleural disease or drug-induced lupus pleuritis. Drugs that may cause lupus pleurisy include *hydralazine, procainamide, and quinidine*.

An 8-year-old male presents with cervical lymphadenitis. He has a kitten at home and you are concerned about cat-scratch disease. Which one of the following antibiotics is most appropriate for treatment of Bartonella henselae infection? A. Azithromycin (Zithromax) B. Ceftriaxone (Rocephin) C. Amoxicillin/clavulanate (Augmentin) D. Doxycycline E. Clindamycin (Cleocin)

*Azithromycin (A)* has been shown to reduce the duration of lymphadenopathy in cat-scratch disease (SOR B). Other antibiotics that have been used include rifampin, ciprofloxacin, trimethoprim/sulfamethoxazole, and gentamicin.

An 8-year-old male presents with cervical lymphadenitis. He has a kitten at home and you are concerned about cat-scratch disease. Which one of the following antibiotics is most appropriate for treatment of Bartonella henselae infection? A. Azithromycin (Zithromax) B. Ceftriaxone (Rocephin) C. Amoxicillin/clavulanate (Augmentin) D. Doxycycline E. Clindamycin (Cleocin)

*Azithromycin (A)* has been shown to reduce the duration of lymphadenopathy in cat-scratch disease. Other antibiotics that have been used include rifampin, ciprofloxacin, trimethoprim/sulfamethoxazole, and gentamicin. Ceftriaxone, amoxicillin/clavulanate, doxycycline, and clindamycin are not effective in the treatment of Bartonella infection.

A 40-year-old female comes to your office with a 1-month history of right heel pain that she describes as sharp, searing, and severe. The pain is worst when she first bears weight on the foot after prolonged sitting and when she gets out of bed in the morning. It gets better with continued walking, but worsens at the end of the day. She does not exercise except for being on her feet all day in the hospital where she works as a floor nurse. She denies any history of trauma. An examination reveals point tenderness to palpation on the plantar surface of the heel at the medial calcaneal tuberosity. Which one of the following should you recommend as first-line treatment? (check one) A. Taping/strapping B. Over-the-counter heel inserts C. Extracorporeal shock wave therapy D. A corticosteroid injection E. A fiberglass walking cast

*B. Over-the-counter heel inserts* C. Extracorporeal shock wave therapy is ineffective D. Corticosteroid injection may have a short-term benefit at one month, but is no better than other treatments at 6 months and carries a risk of plantar fascia rupture. NSAIDs and ice can be used as adjuncts to 1st line therapy. Custom made insoles & night splints and corticosteroid iontophoresis should be reserved as 2nd line therapy for patients who fail 1st line treatment due to their expense. Surgery if conservative therapies fail. Note that it can last for months or years but resolves in most patients over time with or without therapy.

Metformin (Glucophage) should be stopped prior to which one of the following, and withheld until 48 hours after completion of the test? A. An upper GI series B. Abdominal ultrasonography C. CT angiography D. MRI of the brain E. Colonoscopy *Correct.*

*C. CT angiography* Since even a temporary reduction in renal function, such as occurs after pyelography or angiography, can cause lactic acidosis in patients taking metformin, the drug should be discontinued 48 hours before such procedures (SOR C) and restarted 48 hours after the procedure if renal function is normal. The other procedures listed are not indications for stopping metformin.

A 25-year-old medical student reads about the benefits of moderate alcohol consumption on lipid levels and begins to drink 5 ounces of red wine a day, adding 100 calories to his diet. Assuming that his diet and exercise levels stay the same, what effect will the additional 3000 calories a month have on his body weight over the next 10 years? A. They will have essentially no effect B. His weight will increase by about 25 kg C. His weight will increase slightly then stabilize D. His normal caloric expenditure will decrease

*C. His weight will increase slightly then stabilize* There is not a direct relation between daily calorie consumption and weight. An adult male consuming an extra 100 calories a day above his caloric need will not continue to gain weight indefinitely; rather, his weight will increase to a certain point and then become constant. Fat must be fed, and maintaining the newly created tissue requires an increase in caloric expenditure. An extra 100 calories a day will result in a weight gain of approximately 5 kg, which will then be maintained.

As the medical review officer for a local business, you are required to interpret urine drug tests. Assuming the sample was properly collected and handled, which one of the following test results is consistent with the history provided and should be reported as a negative test? A. Diazepam (Valium) identified in an employee taking oxazepam prescribed by a physician B. Morphine identified in an employee undergoing a prescribed methadone pain management program C. Morphine identified in an employee taking a prescribed cough medicine containing codeine D. Tetrahydrocannabinol above the threshold value in an employee who reports secondary exposure to marijuana E. Tetrahydrocannabinol identified in an employee taking prescribed tramadol (Ultram)

*C. Morphine identified in an employee taking a prescribed cough medicine containing codeine* Morphine is a metabolite of codeine. Wrong Answers: A. Oxazepam is a metabolite of diazepam but not the other way around B. Morphine is not a metabolite of methadone D. THC should not be above the threshold due to secondary exposure- that's why there is a threshold in the first place. E. THC is not a metabolite of tramadol. Duh.

A 30-year-old male presents to the emergency department with a sensation of a racing heart. His history is significant for known Wolff-Parkinson-White syndrome (WPW). On examination he is alert and in no severe distress. His blood pressure is 130/70 mm Hg, pulse rate 220 beats/min, and oxygen saturation 96%. An EKG reveals a regular, wide-complex tachycardia with a rate of 220 beats/min. You determine that he is stable, the EKG is consistent with WPW, and pharmacologic conversion is a safe initial therapy. Which one of the following would be the treatment of choice? A. Verapamil (Calan) B. Adenosine (Adenocard) C. Procainamide D. Digoxin

*C. Procainamide* Adenosine, digoxin, and calcium channel antagonists act by blocking conduction through the atrioventricular (AV) node, which may increase the ventricular rate paradoxically, initiating ventricular fibrillation. These agents should be avoided in Wolff-Parkinson-White syndrome. Procainamide is usually the treatment of choice in these situations, although amiodarone may also be used.

A 30-year-old female asks you whether she should have a colonoscopy, as her father was diagnosed with colon cancer at the age of 58. There are no other family members with a history of colon polyps or cancer. You recommend that she have her first screening colonoscopy: A. now and every 5 years if normal B. now and every 10 years if normal C. at age 40 and then every 5 years if normal D. at age 40 and then every 10 years if normal E. at age 50 and then every 5 years if normal

*C. at age 40 and then every 5 years if normal* Patients should be risk-stratified according to their family history. Patients who have one first degree relative diagnosed with colorectal cancer or adenomatous polyps before age 60, or at least two second degree relatives with colorectal cancer, are in the highest risk group. They should start colon cancer screening at age 40, or 10 years before the earliest age at which an affected relative was diagnosed (whichever comes first) and be rescreened every 5 years.

Which one of the following has the best evidence that it is safe for use in pregnancy? A. Alprazolam (Xanax) B. Lithium C. Bupropion (Wellbutrin) D. Fluoxetine (Prozac) E. Paroxetine (Paxil)

*D. Fluoxetine* Lithium is known to be teratogenic. Benzodiazepines such as alprazolam are controversial due to a possible link to cleft lip/palate. Studies have shown no significant risk of congenital anomalies from SSRI use in pregnancy, except for paroxetine. Paroxetine is a category D medication and should be avoided in pregnant women (SOR B). There is concern about an increased risk of congenital cardiac malformations from first-trimester exposure. Bupropion has not been studied extensively for use in pregnancy, and in one published study of 136 patients it was linked to an increased risk of spontaneous abortion.

An 88-year-old male has been hospitalized for the past 3 days after being found on the floor of his home by a neighbor and transported to the hospital by ambulance. He was cachectic and dehydrated at the time of admission, with a serum albumin level of 1.9 g/dL (N 3.5-4.7). He has received intravenous fluids and is now euvolemic. He began nasogastric tube feeding 2 days ago and has now developed nausea, vomiting, hypotension and delirium. Which one of the following is the most classic electrolyte abnormality with this condition? A. Hypocalcemia B. Hypercalcemia C. Hyperkalemia D. Hypophosphatemia E. Hyperphosphatemia

*D. Hypophosphatemia* Refeeding syndrome can occur as a result of the potentially fatal shifts in fluids & electrolytes in malnourished patients receiving artificial refeeding. The hallmark biochemical feature is *hypophosphatemia*. It may also include abnormal sodium & fluid balance; changes in glucose, protein and fat metabolism; thiamine deficiency; hypokalemia; and hypomagnesemia.

A 42-year-old male with a history of intravenous drug use asks to be tested for hepatitis C. The hepatitis C virus (HCV) antibody enzyme immunoassay and recombinant immunoblot assay are both reported as positive. The quantitative HCV RNA polymerase chain reaction test is negative. These test results are most consistent with: (check one) A. very early HCV infection B. current active HCV infection C. a false-positive antibody test D. past infection with HCV that is now resolved *Correct.*

*D. past infection with HCV that is now resolved* The most widely used initial assay for detecting hepatitis C virus (HCV) antibody is the enzyme immunoassay. A positive enzyme immunoassay should be followed by a confirmatory test such as the recombinant immunoblot assay. If negative, it indicates a false-positive antibody test. If positive, the quantitative HCV RNA polymerase chain reaction is used to measure the amount of virus in the blood to distinguish active from resolved HCV infection. In this case, the results of the test indicate that the patient had a past infection with HCV that is now resolved.

A 68-year-old African-American female with primary hypothyroidism is taking levothyroxine (Synthroid), 125 μg/day. Her TSH level is 0.2μU/mL (N 0.5-5.0). She has no symptoms of either hypothyroidism or hyperthyroidism. Which one of the following would be most appropriate at this point? A. Continuing levothyroxine at the same dosage B. Increasing the levothyroxine dosage C. Decreasing the levothyroxine dosage D. Discontinuing levothyroxine E. Ordering a free T 4

*Decreasing the levothyroxine dosage* In a patient receiving levothyroxine, a low TSH usually indicates overreplacement. If this occurs, the dosage should be reduced slightly & the TSH level repeated in 2-3 months.

An 82-year-old resident of a local nursing home is brought to your clinic with fever, difficulty breathing, and a cough productive of purulent sputum. The patient is found to have an oxygen saturation of 86% on room air and a chest radiograph shows a new infiltrate. A decision is made to hospitalize the patient. Which one of the following intravenous antibiotic regimens would be most appropriate for this patient? A. Levofloxacin (Levaquin) B. Ceftriaxone (Rocephin) and azithromycin (Zithromax) C. Ceftazidime (Fortaz, Tazicef) and levofloxacin D. Ceftazidime and vancomycin E. Ceftazidime, levofloxacin, and vancomycin

*E. Ceftazidime, levofloxacin & vancomycin* Nursing home-acquired pneumonia should be suspected in patients with a new infiltrate on a chest radiograph if it is associated with a fever, leukocytosis, purulent sputum, or hypoxia. Nursing-home patients who are hospitalized for pneumonia should be started on intravenous antimicrobial therapy, with empiric coverage for methicillin-resistant Staphylococcus aureus (MRSA) and Pseudomonas aeruginosa. The 2005 American Thoracic Society/Infectious Diseases Society of America guideline recommends combination therapy consisting of an antipseudomonal cephalosporin such as cefepime or ceftazidime, an antipseudomonal carbapenem such as imipenem or meropenem, or an extended-spectrum β-lactam/β-lactamase inhibitor such as piperacillin/tazobactam, PLUS an antipseudomonal fluoroquinolone such as levofloxacin or ciprofloxacin, or an aminoglycoside such as gentamicin, tobramycin, or amikacin, PLUS an anti-MRSA agent (vancomycin or linezolid). Ceftriaxone and azithromycin or levofloxacin alone would be reasonable treatment options for a patient with nursing home-acquired pneumonia who does not require hospitalization.

A 17-year-old white female has a history of anorexia nervosa, and weight loss has recently been a problem. The patient is an academically successful high-school student who lives with her parents and a younger sibling. Her BMI is 17.4 kg/m2 . Her serum electrolyte levels and an EKG are normal. Which one of the following interventions is most likely to be successful? A. Family-based treatment B. Adolescent-focused individual therapy C. Fluoxetine (Prozac) D. Phenelzine (Nardil) E. Desipramine (Norpramin)

*Family-based treatment (A)* for the adolescent with anorexia nervosa has been found to provide superior results when compared with individual adolescent-focused therapy (SOR B). Antidepressants have not been successful. They may be indicated for coexisting conditions, but this is more common with bulimia.

Which one of the following has been shown to be effective for improving symptoms of varicose veins? A. Horse chestnut seed extract B. Vitamin B12 C. Ephedra D. Milk thistle E. St. John's wort

*Horse chestnut seed extract (A)* has been shown to have some effect when used orally for symptomatic treatment of chronic venous insufficiency, such as varicose veins. It may also be useful for relieving pain, tiredness, tension, and swelling in the legs. It contains a number of anti-inflammatory substances, including escin, which reduces edema and lowers fluid exudation by decreasing vascular permeability. Milk thistle may be effective for hepatic cirrhosis. Ephedra is considered unsafe, as it can cause severe life-threatening or disabling adverse effects in some people. St. John's wort may be effective for treating mild to moderate depression. Vitamin B12 is used to treat pernicious anemia.

A 32-year-old white female presents with a 6-week history of increasing headache, which she now describes as severe. The only abnormal finding on examination is a BMI of 32.4 kg/m2 . A neurologic examination is normal. CT of the head is normal and a lumbar puncture is remarkable only for increased cerebrospinal fluid pressure. There is no history of trauma or hypercoagulable disorder. Management should be directed toward preventing which one of the following? A. Visual loss B. Hearing loss C. Vertigo D. A cerebrovascular accident E. Cerebral herniation

*Loss of vision (A)* is a devastating neurologic deficit that occurs with idiopathic intracranial hypertension (pseudotumor cerebri, benign intracranial hypertension), although it is uncommon. Sixth cranial nerve palsies may also occur as a false localizing sign. The typical presentation is a young, obese woman with a headache, palpable tinnitus, and nausea and vomiting. CT is usually normal or shows small ventricles. The lumbar puncture shows elevated pressure with normal fluid examination. CSF protein levels may be low. Hearing loss and vertigo are not characteristic of this disorder.

A 23-year-old Hispanic female at 18 weeks' gestation presents with a 4-week history of a new facial rash. She has noticed worsening with sun exposure. Her past medical history and review of systems is normal. On examination, you note symmetric, hyperpigmented patches on her cheeks and upper lip. The remainder of her examination is normal. The most likely diagnosis is: A. Lupus erythematosus B. Pemphigoid gestationis (herpes gestationis) C. Melasma (chloasma) D. Prurigo gestationis

*Melasma or chloasma (C)* is common in pregnancy, with approximately 70% of pregnant women affected. It is an acquired hypermelanosis of the face, with symmetric distribution usually on the cheeks, nose, eyebrows, chin, and/or upper lip. The pathogenesis is not known. UV sunscreen is important, as sun exposure worsens the condition. Melasma often resolves or improves post partum. Persistent melasma can be treated with hydroquinone cream, retinoic acid, and/or chemical peels performed post partum by a dermatologist. The facial rash of lupus is usually more erythematous, and lupus is relatively rare. Pemphigoid gestationis is a rare autoimmune disease with extremely pruritic, bullous skin lesions that usually spare the face. Prurigo gestationis involves pruritic papules on the extensor surfaces and is usually associated with significant excoriation by the uncomfortable patient.

A 56-year-old male with type 2 diabetes mellitus has normal cardiac and renal function but has failed to achieve adequate control of his diabetes with diet and multiple oral agents. His BMI is 30.1 kg/m2 and his hemoglobin A1c level is 9.1%. Which one of the following is most likely to be beneficial in combination with insulin and diet therapy in this patient? A. Acarbose (Precose) B. Glimepiride (Amaryl) C. Metformin (Glucophage) D. Pioglitazone (Actos) E. Repaglinide (Prandin)

*Metformin (C)* has been found to reduce cardiovascular risk in patients with type 2 diabetes mellitus. It also decreases the risk of weight gain, and unlike some oral agents it does not significantly increase the risk of hypoglycemia. It should be continued when insulin is initiated in patients with no renal impairment.

A 62-year-old male with a history of prostate cancer and well-controlled hypertension presents with severe osteoporosis. At 55 years of age he received prostate brachytherapy and androgen deprivation for his prostate cancer and has been disease-free since. He presently takes lisinopril (Prinivil, Zestril), 5 mg daily; alendronate (Fosamax), 70 mg weekly; calcium, 1000 mg daily; and vitamin D, 1200 units daily. He has never smoked, exercises five times a week, and maintains a healthy lifestyle. In spite of his lifestyle and the medications he takes, he continues to have severe osteoporosis on his yearly bone density tests. In addition to recommending fall precautions, which one of the following would you consider next to treat his osteoporosis? A. Testosterone B. Calcitonin C. Teriparatide (Forteo) D. Raloxifene (Evista) E. Zoledronic acid (Reclast)

*Teriparatide (C)* is indicated for the treatment of severe osteoporosis, for patients with multiple osteoporosis risk factors, or for patients with failure of bisphosphonate therapy (SOR B). Therapy with teriparatide is currently limited to 2 years and is contraindicated in patients with a history of bone malignancy, Paget disease, hypercalcemia, or previous treatment with skeletal radiation. Its route of administration (subcutaneous) and high cost should be considered when prescribing teriparatide therapy. Testosterone therapy is contraindicated in patients with a history of prostate cancer. Zoledronic acid is a parenterally administered bisphosphonate and would not be appropriate in a patient who has already failed bisphosphonate therapy. Likewise, raloxifene and calcitonin are not indicated in patients with severe osteoporosis who have failed bisphosphonate therapy.

A previously healthy 67-year-old male sees you for a routine health maintenance visit. During the physical examination you discover a harsh systolic murmur that is loudest over the second right intercostal space and radiates to the carotid arteries. The patient denies any symptoms of dyspnea, angina, syncope, or decreased exertional tolerance. An echocardiogram shows severe aortic stenosis, with an aortic valve area of <1 cm 2, a mean gradiant >40 mm Hg, and an ejection fraction of 60%. Which one of the following would be most appropriate at this point? A. Coronary angiography B. Exercise stress testing C. Treatment with prazosin (Minipress) D. Referral for aortic valve replacement E. Watchful waiting

*Watchful waiting (E)* is recommended for most patients with asymptomatic aortic stenosis, including those with severe disease. This is because the surgical risk of aortic valve replacement outweighs the approximately 1% annual risk of sudden death in asymptomatic patients with aortic stenosis. Peripheral α-blockers, such as prazosin, should be avoided because of the risk of hypotension or syncope. Coronary angiography should be reserved for symptomatic patients who do not have evidence of severe aortic stenosis on echocardiography performed to evaluate their symptoms, or for preoperative evaluation prior to aortic valve replacement. Exercise stress testing is not safe with severe aortic stenosis because of the risk of death during the test.

A 42-year-old African-American male recently traveled to the Caribbean for a scuba diving trip. Since his return he has noted brief intermittent episodes of vertigo not associated with nausea or vomiting. He is concerned, however, because these episodes occurred after sneezing or coughing and then a couple of times after straining while lifting something. He has had no hearing loss, and no vertigo with positional changes such as bending over or turning over in bed. The most likely cause of this patients vertigo is A. vestibular neuronitis B. Menieres disease C. benign paroxysmal positional vertigo D. a perilymphatic fistula E. multiple sclerosis triggered by a rapid change in climate

A *perilymphatic fistula (D)* between the middle and inner ear may be caused by barotrauma from scuba diving, as well as by direct blows, heavy weight bearing, and excessive straining (e.g., with sneezing or bowel movements.) This patients recent trip involved two of these potential factors. Vestibular neuronitis is a more sudden, unremitting syndrome. Menieres disease is manifested by episodes of vertigo, associated with hearing loss and often with nausea and vomiting. Benign paroxysmal positional vertigo is more likely in older individuals, and is associated with postural change. Multiple sclerosis requires symptoms in multiple areas and is not thought to be provoked by climate change.

A 48-year-old white female comes to see you because of abnormal vaginal bleeding. Her periods are lasting 3-5 days longer than usual, bleeding is heavier, and she has experienced some intermenstrual bleeding. Her physical examination is unremarkable, except for a parous cervix with dark blood at the os and in the vagina. She has no orthostatic hypotension, and her hemoglobin level is 11.5 g/dL. A pregnancy test is negative. Which one of the following is the most important next step in management? A. Laboratory tests to rule out thyroid dysfunction B. An endometrial biopsy C. Oral contraceptives, 4 times a day for 5-7 days D. Cyclic combination therapy with conjugated estrogens (Premarin) and medroxy-progesterone (Provera) each month E. Administration of a gonadotropin-releasing hormone analog such as leuprolide acetate (Eligard Lupron Depot)

A patient over the age of 35 who experiences abnormal vaginal bleeding must have an endometrial assessment to exclude endometrial hyperplasia or cancer. An *endometrial biopsy (B)* is currently the preferred method for identifying endometrial disease. A laboratory evaluation for thyroid dysfunction or hemorrhagic diathesis is appropriate if no cancer is present on an endometrial biopsy and medical therapy fails to halt the bleeding. The other options listed can be used as medical therapy to control the bleeding once the histopathologic diagnosis has been made.

A painful thrombosed external hemorrhoid diagnosed within the first 24 hours after occurrence is ideally treated by: A. appropriate antibiotics B. office banding C. office cryotherapy D. thrombectomy under local anesthesia E. total hemorrhoidectomy

A thrombosed external hemorrhoid is manifested by the sudden development of a painful, tender, perirectal lump. Because there is somatic innervation, the pain is intense, and increases with edema. Treatment involves *excision of the acutely thrombosed tissue under local anesthesia (D)*, mild pain medication, and sitz baths. It is inappropriate to use procedures that would increase the pain, such as banding or cryotherapy. Total hemorrhoidectomy is inappropriate and unnecessary.

A 72-year-old male with a history of hypertension and a previous myocardial infarction is diagnosed with heart failure. Echocardiography reveals systolic dysfunction, and recent laboratory tests indicated normal renal function, with a serum creatinine level of 1.1 mg/dL (N <1.5), a sodium level of 139 mEq/L (N 136-145), and a potassium level of 3.5 mEq/L (N 3.5-5.0). He is currently asymptomatic. Which one of the following medications would be the best choice for initial management in this patient? A. Furosemide (Lasix) B. Isosorbide dinitrate (Isordil) C. Spironolactone (Aldactone) D. Digoxin E. Lisinopril (Prinivil, Zestril)

ACE inhibitors such as *lisinopril (E)* are indicated for all patients with heart failure due to systolic dysfunction, regardless of severity. ACE inhibitors have been shown to reduce both morbidity and mortality, in both asymptomatic and symptomatic patients, in randomized, controlled trials. *Unless absolutely contraindicated, ACE inhibitors should be used in all heart failure patients.* No ACE inhibitor has been shown to be superior to another, and no study has failed to show benefit from an ACE inhibitor. Direct-acting vasodilators such as isosorbide dinitrate also could be used in this patient, but ACE inhibitors have been shown to be superior in randomized, controlled trials. β-Blockers are also recommended in heart failure patients with systolic dysfunction, except those who have dyspnea at rest or who are hemodynamically unstable. These agents have been shown to reduce mortality from heart failure. A diuretic such as furosemide may be indicated to relieve congestion in symptomatic patients. Aldosterone antagonists such as spironolactone are also indicated in patients with symptomatic heart failure. Digoxin currently is recommended for patients with heart failure and atrial fibrillation, and can be considered in patients who continue to have symptoms despite maximal therapy with other agents.

Screening for osteoporosis should be done in which one of the following groups? A. Postmenopausal women B. Women over age 50 with a BMI ≥30 kg/m2 C. Men over age 50 with type 2 diabetes mellitus D. Men over age 70

All women ≥65 and all men ≥70 should be screened for osteoporosis. For men and women age 50-69, the presence of factors associated with low bone density would merit screening. Risk factors include low body weight, previous fracture, a family history of osteoporosis with fracture, a history of falls, physical inactivity, low vitamin D or calcium intake, and the use of certain medications or the presence of certain medical conditions. Chronic systemic diseases that increase risk include COPD, HIV, severe liver disease, renal failure, systemic lupus erythematosus, and rheumatoid arthritis. Endocrine disorders that increase risk include type 1 diabetes mellitus, hyperparathyroidism, hyperthyroidism, Cushing's syndrome, and others. Medications that increase risk include anticonvulsants, corticosteroids, and immunosuppressants. Nutritional risks include celiac disease, vitamin D deficiency, anorexia nervosa, gastric bypass, and increased alcohol or caffeine intake.

A 62-year-old male has been taking omeprazole (Prilosec) for over a year for gastroesophageal reflux disease. He is asymptomatic and has had no problems tolerating the drug, but asks you about potential side effects, as well as the benefits of continuing therapy. It would be most accurate to tell him that omeprazole therapy is associated with which one of the following? A. A decreased rate of hip fracture B. Decreased vitamin B12 absorption C. A reduced likelihood of pneumonia D. A reduced likelihood of Clostridium difficile colitis E. An increased likelihood of iron deficiency anemia

Although proton pump inhibitors are the most effective treatment for patients with asymptomatic gastroesophageal reflux disease, there are several potential problems with prolonged therapy. Omeprazole is associated with an increased risk of community-acquired pneumonia and Clostridium difficile colitis. Omeprazole has also been shown to acutely *decrease the absorption of vitamin B12 (B)* , and it decreases calcium absorption, leading to an increased risk of hip fracture.

You are asked to perform a preoperative evaluation on a 55-year-old white female with type 2 diabetes mellitus prior to elective femoral-anterior tibial artery bypass surgery. She is unable to climb a flight of stairs or do heavy work around the house. She denies exertional chest pain, and is otherwise healthy. Based on current guidelines, which one of the following diagnostic studies would be appropriate prior to surgery because the results could alter the management of this patient? A. Pulmonary function studies B. Coronary angiography C. Carotid angiography D. A dipyridamole-thallium scan E. A hemoglobin A1c level

Answer: *D. Dipyridamole-thallium scan* (myocardial perfusion study - dipyridamole causes coronary vasoconstrition & thallium is the radioisotope) This patient is not undergoing emergency surgery, nor does she have an active cardiac condition; however, she is undergoing a high-risk procedure (>5% risk of perioperative myocardial infarction) with vascular surgery. As she cannot climb a flight of stairs or do heavy housework, her functional status is <4 METs, and she should be considered for further evaluation. The patient's diabetes is an additional clinical risk factor. With vascular surgery being planned, appropriate recommendations include proceeding with the surgery with heart rate control, or performing noninvasive testing if it will change the management of the patient. Coronary angiography is indicated if the noninvasive testing is abnormal. Pulmonary function studies are most useful in patients with underlying lung disease or those undergoing pulmonary resection. Hemoglobin A1c is a measure of long-term diabetic control and is not particularly useful perioperatively. Carotid angiography is not indicated in asymptomatic patients being considered for lower-extremity vascular procedures.

A 72-year-old male with COPD presents to the emergency department with an acute exacerbation marked by increased sputum production and shortness of breath. His oxygen saturation is 88% on room air and he has diffuse inspiratory and expiratory wheezes bilaterally. In addition to oxygen and bronchodilators, which one of the following is most appropriate for this patient? A. No additional treatments B. Systemic corticosteroids only C. Inhaled corticosteroids only D. Systemic corticosteroids and antibiotics E. Inhaled corticosteroids and antibiotics

Answer: *Systemic corticosteroids & antibiotics (D)* Acute exacerbations of COPD are very common, with most caused by superimposed infections. Supplemental oxygen, antibiotics, and bronchodilators are used for management. Systemic corticosteroids, either oral or parenteral, have been shown to significantly reduce treatment failures and improve lung function and dyspnea over the first 72 hours, although there is an increased risk of adverse drug reactions.

Which one of the following is most commonly implicated in interstitial nephritis? A. NSAIDs B. ACE inhibitors C. Diuretics D. Corticosteroids E. Antibiotics

Antibiotics, especially penicillins, cephalosporins, and sulfonamides, are the most common drug-related cause of acute interstitial nephritis. Corticosteroids may be useful for treating this condition. The other drugs listed may cause renal injury, but not acute interstitial nephritis.

A 45-year-old male sees you for a routine annual visit and is found to have atrial fibrillation, with a ventricular rate of 70-75 beats/min. He is otherwise healthy, and a laboratory workup and echocardiogram are normal. Which one of the following would be the most appropriate management? A. Aspirin, 325 mg daily B. Warfarin (Coumadin), with a target INR of 2.0-3.0 C. Clopidogrel (Plavix), 75 mg daily D. Amiodarone (Cordarone), 200 mg daily E. Observation only

Atrial fibrillation is the most common arrhythmia, and its prevalence increases with age. The major risk with atrial fibrillation is stroke, and a patient's risk can be determined by the CHADS 2 score. CHADS stands for Congestive heart failure, Hypertension, Age >75, Diabetes mellitus, and previous Stroke or transient ischemic attack. Each of these is worth 1 point except for stroke, which is worth 2 points. A patient with 4 or more points is at high risk, and 2-3 points indicates moderate risk. Having ≤1 point indicates low risk, and this patient has 0 points. Low-risk patients should be treated with *aspirin, 81-325 mg daily (A)*. Moderate-or high-risk patients should be treated with warfarin.

A 55-year-old male presents with a 2-year history of persistent, worsening neck stiffness. Over the past month, the stiffness has been associated with left thumb tingling. After completing a thorough history and physical examination, which one of the following studies would be the most appropriate next step in further evaluating the patient's complaints? A. Lateral neck radiography B. A cervical spine series C. Neck MRI D. CT myelography E. Diskography

Based on the American College of Radiology Appropriateness Criteria for chronic neck pain, a *complete cervical spine series (B)* that includes five views is the correct study in a patient of any age with chronic neck pain and no history of trauma, malignancy, or surgery. If the radiographs are normal and the patient has neurologic signs or symptoms, the next step would be MRI. If MRI is contraindicated, CT myelography should be offered.

A 14-year-old female with a history of asthma is having daytime symptoms about once a week and symptoms that awaken her at night about once a month. Her asthma does not interfere with normal activity, and her FEV1 is >80% of predicted. Which one of the following is the most appropriate treatment plan for this patient? *A. A short-acting inhaled β-agonist as needed* B. Low-dose inhaled corticosteroids daily C. A leukotriene receptor antagonist daily D. Medium-dose inhaled corticosteroids daily E. Low-dose inhaled corticosteroids plus a long-acting inhaled β-agonist daily

Based on this patient's reported frequency of asthma symptoms, she should be classified as having intermittent asthma. The preferred first step in managing intermittent asthma is an inhaled short-acting β-agonist as needed. Daily medication is reserved for patients with persistent asthma (symptoms >2 days per week for mild, daily for moderate, and throughout the day for severe) and is initiated in a stepwise approach, starting with a daily low-dose inhaled corticosteroid or leukotriene receptor antagonist and then progressing to a medium-dose inhaled corticosteroid or low-dose inhaled corticosteroid plus a long-acting inhaled β-agonist.

A 63-year-old white male sees you for an initial visit and is accompanied by his daughter, who is a patient of yours and scheduled the visit. The father recently relocated to be near the daughter after his wife died. He has well-controlled type 2 diabetes mellitus, but is otherwise healthy. Referring to the copy of the medical records they brought with them, the daughter notes that her father has received influenza vaccine in 3 of the past 5 years, but she can find no documentation that he ever had "the pneumonia vaccine." She asks if he should receive it at this visit. You advise them that he should receive pneumococcal vaccine: A. annually, along with influenza vaccine B. now and a repeat dose every 5 years C. every 5 years starting at age 65 *D. now and a repeat dose once at age 68* E. only once, at age 65

Both the CDC and the American Academy of Family Physicians recommend that *all adults over the age of 65 receive a single dose of pneumococcal polysaccharide vaccine*. Immunization before the age of 65 is recommended for certain subgroups of adults, including institutionalized individuals over the age of 50; those with chronic cardiac or pulmonary disease, diabetes mellitus, anatomic asplenia, chronic liver disease, or kidney failure; and health-care workers. It is *recommended that those receiving the vaccine before the age of 65 receive an additional dose at age 65 or 5 years after the first dose, whichever is later.*

Breastfeeding a full-term, healthy infant is contraindicated when which one of the following maternal conditions is present? A. Chronic hepatitis B infection B. Seropositive cytomegalovirus carrier state C. Current tobacco smoking D. Herpes simplex viral lesions on the breasts E. Undifferentiated fever

Breastfeeding provides such optimal nutrition for an infant that the benefits still far outweigh the risks even when the mother smokes tobacco, tests positive for hepatitis B or C virus, or develops a simple undifferentiated fever. Maternal seropositivity to cytomegalovirus (CMV) is not considered a contraindication except when it has a recent onset or in mothers of low birthweight infants. When present, the CMV load can be substantially reduced by freezing and pasteurization of the milk. All patients who smoke should be strongly encouraged to discontinue use of tobacco, particularly in the presence of infants, but smoking is not a contraindication to breastfeeding. Mothers with active *herpes simplex lesions on a breast (D)* should not feed their infant from the infected breast, but may do so from the other breast if it is not infected. Breastfeeding is also contraindicated in the presence of active maternal tuberculosis, and following administration or use of radioactive isotopes, chemotherapeutic agents, "recreational" drugs, or certain prescription drugs.

A 42-year-old male with well-controlled type 2 diabetes mellitus presents with a 24-hour history of influenza-like symptoms, including the sudden onset of headache, fever, myalgias, sore throat, and cough. It is December, and there have been a few documented cases of influenza recently in the community. The CDC recommends initiating treatment in this situation: *A. on the basis of clinical symptoms alone* B. only if rapid influenza testing is positive C. only if the diagnosis is confirmed by immunoassay testing D. only if the diagnosis is confirmed by reverse transcriptase polymerase chain reaction (PCR) assay

CDC Recommendations: 1. Antiviral treatment is recommended as soon as possible for patients with confirmed or suspected influenza who have severe, complicated, or progressive illness or who require hospitalization. 2. Antiviral treatment is recommended as soon as possible for outpatients with confirmed or suspected influenza who are at higher risk for influenza complications based on their age or underlying medical conditions. Clinical judgment should be an important component of outpatient treatment decisions. 3. Antiviral treatment also may be considered on the basis of clinical judgment for any outpatient with confirmed or suspected influenza who does not have known risk factors for severe illness, if treatment can be initiated within 48 hours of illness onset. Many rapid influenza tests produce false-negative results, and more accurate assays can take more than 24 hours. Thus, treatment of patients with a clinical picture suggesting influenza is recommended, even if a rapid test is negative. Delaying treatment until further test results are available is not recommended.

The most appropriate advice for a 50-year-old female who has passed six calcium oxalate stones over the past 4 years is to: A. restrict her calcium intake B. restrict her intake of yellow vegetables C. increase her sodium intake D. increase her dietary protein intake E. take potassium citrate with meals

Calcium oxalate stones are the most common of all renal calculi. A low-sodium, restricted-protein diet with increased fluid intake reduces stone formation. A low-calcium diet has been shown to be ineffective. Oxalate restriction also reduces stone formation. Oxalate-containing foods include spinach, chocolate, tea, and nuts, but not yellow vegetables. *Potassium citrate should be taken at mealtime (E)* to increase urinary pH and urinary citrate.

Which one of the following is consistent with spinal stenosis but not with a herniated vertebral disk? A. Numbness B. Muscle weakness *C. Pain relieved by sitting* D. Pain relieved by standing

Causes of low back pain include vertebral disk herniation and spinal stenosis. Numbness and muscle weakness may be present in both. Pain from spinal stenosis is relieved by sitting and aggravated by standing, whereas the opposite is true for pain from a herniated disk.

A middle-aged hairdresser presents with a complaint of soreness of the proximal nail folds of several fingers on either hand, which has slowly worsened over the last 6 months. The nails appear thickened and distorted. Otherwise she is healthy and has no evidence of systemic disease. Which one of the following would be the most effective initial treatment? A. Soaking in a dilute iodine solution twice daily to cleanse and sterilize the nail beds B. Oral amoxicillin/clavulanate (Augmentin) for up to 4-6 weeks C. Topical betamethasone dipropionate (Diprolene) applied twice daily to the nail folds for 3-4 weeks D. Evaluation for HIV, hepatitis C, psoriasis, and rheumatoid arthritis

Chronic paronychia is a common condition in workers whose hands are exposed to chemical irritants or are wet for long periods of time. This patient is an otherwise healthy hairdresser, with frequent exposure to irritants. The patient should be advised to avoid exposure to harsh chemicals and water. In addition, the use of strong *topical corticosteroids over several weeks (C)* can greatly reduce the inflammation, allowing the nail folds to return to normal and helping the cuticles recover their natural barrier to infection. Soaking in iodine solution would kill bacteria, but would also perpetuate the chronic irritation. Because the condition is related to chemical and water irritation, a prolonged course of antibiotics should not be the first treatment step, and could have serious side effects.

A 40-year-old female with chronic plaque psoriasis requests topical treatment. Which one of the following topical therapies would be most effective and have the fewest adverse effects? A. High-potency corticosteroids B. Tazarotene (Tazorac) C. Coal tar polytherapy D. Anthralin

Chronic plaque psoriasis is the most common type of psoriasis and is characterized by redness, thickness, and scaling. A variety of treatments were found to be more effective than placebo, but the best results were produced by topical vitamin D analogues and topical corticosteroids. Vitamin D and *high-potency corticosteroids (A)* were equally effective when compared head to head, but the corticosteroids produced fewer local reactions (SOR A).

A 28-year-old female consults you because of fatigue, arthralgias that are worse in the morning, and painful, swollen finger joints. She is a high-school teacher. Her erythrocyte sedimentation rate is 60 mm/hr and a test for rheumatoid factor is strongly positive. The best choice for initial therapy would be: A. prednisone B. aspirin C. naproxen (Naprosyn) D. rituximab (Rituxan) E. methotrexate (Rheumatrex)

Disease-modifying drugs such as *methotrexate (E)* are the best choice for initial therapy. Aspirin and NSAIDs are no longer considered first-line treatment because of concerns about their limited effectiveness, inability to modify the long-term course of the disease, and gastrointestinal and cardiotoxic effects. Glucocorticoids such as prednisone are often useful, but have significant side effects. Biologic agents such as rituximab are expensive and have significantly more side effects than methotrexate.

A 55-year-old female who has hypertension, hyperlipidemia, and osteoarthritis of the knees develops acute gout and is found to have hyperuricemia. Discontinuation of which one of the following medications may improve her hyperuricemia? A. Hydrochlorothiazide B. Losartan (Cozaar) C. Metoprolol (Lopressor) D. Simvastatin (Zocor) E. Acetaminophen

Diuretics such as hydrochlorothiazide are known to increase serum uric acid levels, but losartan has been shown to decrease uric acid. Metoprolol, simvastatin, and acetaminophen have no specific effect on serum uric acid levels.

What is the most common cause of erythema multiforme, accounting for more than 50% of cases? A. Candida albicans B. Herpes simplex virus C. Mycoplasma pneumoniae D. Penicillin therapy E. Sulfonamide therapy

Erythema multiforme usually occurs in adults 20-40 years of age, although it can occur in patients of all ages. *Herpes simplex virus (B)* is the most commonly identified cause of this hypersensitivity reaction, accounting for more than 50% of cases.

A 23-year-old white male is brought to the emergency department with slurred speech, confusion, and ataxia. He works as an auto mechanic and has been known to consume alcohol heavily in the past, but denies recent alcohol intake. He appears intoxicated, but no odor of alcohol is noted on his breath. Abnormalities on the metabolic profile include a carbon dioxide content of 10 mmol/L (N 20-30). His blood alcohol level is <10 mg/dL (0.01%). A urinalysis shows calcium oxalate crystals and an RBC count of 10-20/hpf. Woods lamp examination of the urine shows fluorescence. His arterial pH is 7.25. Which one of the following would be most appropriate at this point? (check one) A. Immediate hemodialysis B. Gastric lavage C. Administration of activated charcoal D. Fomepizole (Antizol)

Ethylene glycol poisoning should be suspected in patients with metabolic acidosis of unknown cause and subsequent renal failure, as rapid diagnosis and treatment will limit the toxicity and decrease both morbidity and mortality. This diagnosis should be considered in a patient who appears intoxicated but does not have an odor of alcohol, and has anion gap acidosis, hypocalcemia, urinary crystals, and nontoxic blood alcohol levels. Ethylene glycol is found in products such as engine coolant, de-icing solution, and carpet and fabric cleaners. Until recently, ethylene glycol poisoning was treated with sodium bicarbonate, ethanol, and hemodialysis. Treatment with *fomepizole (Antizol) (D)* has this specific indication, however, and should be initiated immediately when ethylene glycol poisoning is suspected. If ethylene glycol poisoning is treated early, hemodialysis may be avoided, but once severe acidosis and renal failure have occurred hemodialysis is necessary. Ethylene glycol is rapidly absorbed, and use of ipecac or gastric lavage is therefore not effective. Large amounts of activated charcoal will only bind to relatively small amounts of ethylene glycol, and the therapeutic window for accomplishing this is less than 1 hour.

The FDA issued a boxed warning describing an increased risk of tendinopathy and tendon rupture associated with the use of which class of antibiotics? (check one) A. Macrolides B. Aminoglycosides *C. Fluoroquinolones* D. Tetracyclines E. Polypeptides

Fluoroquinolones are associated with an increased risk of tendinopathy and tendon rupture. About 1/6000 prescriptions will cause an Achilles tendon rupture. The risk is higher in those also taking corticosteroids or over the age of 60.

You are initiating treatment for a patient being admitted to the hospital with a new diagnosis of pulmonary embolus. Low molecular weight heparin and warfarin (Coumadin) are started immediately. When can the low molecular weight heparin be stopped? A. When the INR is ≥2.0 B. When the INR is ≥2.0 for 24 hours C. After 4 days, if the INR is ≥³2.0 D. After 4 days, if the INR has been ≥2.0 for 24 hours E. After 5 days, if the INR has been ≥2.0 for 24 hours

For patients with a pulmonary embolus, American College of Chest Physicians guidelines recommend initial treatment with *LMWH, unfractionated heparin, or fondaparinux for at least 5 days, and then can be stopped if the INR has been ≥2.0 for at least 24 hours (E)*. Warfarin reduces the activity of coagulation factors II, VII, IX, and X produced in the liver. Coagulation factors produced prior to initiating warfarin remain active for their usual several-day lifespan, which is why LMWH and warfarin must be given concomitantly for at least 5 days. The INR may reach levels >2.0 before coagulation factors II and X have reached their new plateau levels, accounting for the need for an additional 24 hours of combined therapy before stopping LMWH.

Which one of the following should be used first for ventricular fibrillation when an initial defibrillation attempt fails? A. Amiodarone (Cordarone) B. Lidocaine (Xylocaine) C. Adenosine (Adenocard) D. Vasopressin (Pitressin) E. Magnesium

For persistent ventricular fibrillation (VF), in addition to electrical defibrillation and CPR, patients should be given a vasopressor, which can be either epinephrine or *vasopressin (D)*. Vasopressin may be substituted for the first or second dose of epinephrine. Amiodarone should be considered for treatment of VF unresponsive to shock delivery, CPR, and a vasopressor. Lidocaine is an alternative antiarrhythmic agent, but should be used only when amiodarone is not available. Magnesium may terminate or prevent torsades de pointes in patients who have a prolonged QT interval during normal sinus rhythm. Adenosine is used for the treatment of narrow complex, regular tachycardias and is not used in the treatment of ventricular fibrillation.

A previously healthy 82-year-old male is brought to your office by his daughter after a recent fall while getting up to go to the bathroom in the middle of the night. The patient denies any history of dizziness, chest pain, palpitations, or current injury. He has a history of bilateral dense cataracts. On examination, he is found to have an increased stance width and walks carefully and cautiously with his arms and legs abducted. A timed up-and-go test is performed, wherein the patient is asked to rise from a chair without using his arms, walk 3 meters, turn, return to his chair, and sit down. It takes the patient 25 seconds and he is noted to have an "en bloc" turn. Which one of the following is the most likely cause of this patient's gait and balance disorder? A. Visual impairment B. Cerebellar degeneration C. Frontal lobe degeneration D. Parkinson's disease E. Motor neuropathy

Gait and balance disorders are one of the most common causes of falls in older adults. Correctly identifying gait and balance disorders helps guide management and may prevent consequences such as injury, disability, loss of independence, or decreased quality of life. The "Timed Up and Go" test is a reliable diagnostic tool for gait and balance disorders and is quick to administer. A time of <10 seconds is considered normal, a time of >14 seconds is associated with an increased risk of falls, and a time of >20 seconds usually suggests severe gait impairment. This patient has the cautious gait associated with *visual impairment (A)*. It is characterized by abducted arms and legs; slow, careful, "walking on ice" movements; a wide-based stance; and "en bloc" turns. Patients with cerebellar degeneration have an ataxic gait that is wide-based and staggering. Frontal lobe degeneration is associated with gait apraxia that is described as "magnetic," with start and turn hesitation and freezing. Parkinson's disease patients have a typical gait that is short-stepped and shuffling, with hips, knees, and spine flexed, and may also exhibit festination and "en bloc" turns. Motor neuropathy causes a "steppage" gait resulting from foot drop with excessive flexion of the hips and knees when walking, short strides, a slapping quality, and frequent tripping.

You see a newly adopted 5-month-old for his first well child visit. The parents ask when the child can sit in a safety seat in the car facing forward. You would advise that the child should face rearward until he is at least: (check one) *A. 12 months of age AND weighs 20 lb* B. 15 months of age AND weighs 25 lb C. 15 months of age OR weighs 25 lb D. 18 months of age AND weighs 30 lb E. 18 months of age OR weighs 30 lb

If a child faces forward in a crash, the force is distributed via the harness system across the shoulders, torso, and hips, but the head and neck have no support. Without support, the infant's head moves rapidly forward in flexion while the body stays restrained, causing potential injury to the neck, spinal cord, and brain. In a rear-facing position, the force of the crash is distributed evenly across the baby's torso, and the back of the child safety seat supports and protects the head and neck. For these reasons, the rear-facing position should be used until the child is at least 12 months old and weighs at least 20 lb (9 kg). For example, a 13-month-old child who weighs 19 lb should face rearward, and a 6-month-old child who weighs 21 lb should also face rearward.

The most appropriate initial treatment for scabies in an 8-year-old male is: A. 0.5% malathion lotion (Ovide) B. 5% permethrin cream (Elimite) C. 5% precipitated sulfur in petroleum D. trimethoprim/sulfamethoxazole (Bactrim, Septra) orally for 10 days

In adults and children over 5 years of age, *5% permethrin cream (B)* is standard therapy for scabies. This agent is highly effective, minimally absorbed, and minimally toxic.

A previously healthy gravida 1 para 1 who is 3 weeks post partum complains of bilateral nipple pain with breastfeeding. When she first started breastfeeding she had some soreness that went away after repositioning with feeding. The current pain began gradually 3 days ago. It has been worsening, inhibiting feeding, and is present between feedings. Examination of the breast is notable for erythema and cracking of the areola. The most likely cause is: A. engorgement B. mastitis C. improper latch-on D. eczema flare E. Candida infection

In breastfeeding women, bilateral nipple pain with and between feedings after initial soreness has resolved is usually due to *Candida infection (E)*. Pain from engorgement typically resolves after feeding. Mastitis is usually unilateral and is associated with systemic symptoms and wedge-shaped erythema of the breast tissue. Improper latch-on is painful only during feedings. Eczema isolated to the nipple, while a reasonable part of the differential, would be much more unusual.

A 60-year-old male has a drug-eluting stent placed in his right coronary artery. He will require treatment to prevent stent thrombosis, and once his initial treatment period is completed he will be placed on aspirin, 75-165 mg/day indefinitely. Which one of the following is the preferred initial regimen for preventing stent thrombosis in this situation? A. Aspirin/dipyridamole (Aggrenox) for 3 months B. Aspirin, 162-325 mg/day for 3 months C. Aspirin, 162-325 mg/day, plus clopidogrel (Plavix), both for 3 months D. Aspirin, 162-325 mg/day, plus clopidogrel, both for 12 months E. Warfarin (Coumadin) for 3 months

In patients with a drug-eluting stent, *combined therapy with clopidrogel and aspirin is recommended for 12 months (D)* because of the increased risk of late stent thrombosis. After this time, aspirin at a dosage of 75-165 mg/day is recommended. The minimum duration of combined therapy is 1 month for a bare metal stent, 3 months for a sirolimus-eluting stent, and 6 months for other drug-eluting stents.

A 59-year-old male with known cirrhosis is beginning to show some lower abdominal distention. Ultrasonography confirms your suspicion that he has developed moderate ascites for the first time. Which one of the following is recommended as the initial treatment of choice for this condition? A. Chlorthalidone B. Spironolactone (Aldactone) C. Furosemide (Lasix) D. Ramipril (Altace) E. Large-volume paracentesis

In patients with grade 2 ascites (visible clinically by abdominal distention, not just with ultrasonography), the initial treatment of choice is diuretics along with salt restriction. Aldosterone antagonists such as *spironolactone (B)* are more effective than loop diuretics such as furosemide. Chlorthalidone, a thiazide diuretic, is not recommended. Large-volume paracentesis is the recommended treatment of grade 3 ascites (gross ascites with marked abdominal distention), and is followed by salt restriction and diuretics.

A 65-year-old female who is morbidly obese presents to your office with intertrigo in the axilla. On examination you detect small, reddish-brown macules that are coalescing into larger patches with sharp borders. You suspect cutaneous erythrasma complicating the intertrigo. What would be the most appropriate topical treatment for this condition? A. Cornstarch B. A mild corticosteroid lotion C. A high-potency corticosteroid lotion D. Erythromycin

Intertrigo is inflammation of skinfolds caused by skin-on-skin friction and is common on opposing cutaneous or mucocutaneous surfaces. Secondary cutaneous bacterial and fungal infections are common complications. Cutaneous erythrasma may complicate intertrigo of interweb areas, intergluteal and crural folds, axillae, or inframammary regions. Erythrasma is caused by Corynebacterium minutissimum and presents as small reddish-brown macules that may coalesce into larger patches with sharp borders. Intertrigo complicated by erythrasma is treated with topical or oral *erythromycin (D)*.

A 67-year-old white male with hypertension and chronic kidney disease presents with the recent onset of excessive thirst, frequent urination, and blurred vision. Laboratory testing reveals a fasting blood glucose level of 270 mg/dL, a hemoglobin A 1c of 8.5%, a BUN level of 32 mg/dL, and a serum creatinine level of 2.3 mg/dL. His calculated glomerular filtration rate is 28 mL/min. Which one of the following medications should you start at this time? A. Glipizide (Glucotrol) B. Metformin (Glucophage) C. Glyburide (DiaBeta) D. Acarbose (Precose)

It is recommended that metformin be avoided in patients with a creatinine level >1.5 mg/dL for men or >1.4 mg/dL for women. Glyburide has an active metabolite that is eliminated renally. This metabolite can accumulate in patients with chronic kidney disease, resulting in prolonged hypoglycemia. Acarbose should be avoided in patients with chronic kidney disease, as it has not been evaluated in these patients. *Glipizide (A)* does not have an active metabolite, and is safe in patients with chronic renal disease.

You have been treating a 43-year-old male for unipolar depression for 4 years. He has developed treatment-resistant depression, and despite having a good initial response to an SSRI, his symptoms are worsening. He has failed to improve despite escalated doses of multiple SSRIs and SNRIs. He is currently taking citalopram (Celexa), 60 mg daily. Of the following, the most effective adjunctive therapy would be augmentation with: A. lithium bicarbonate B. high-dose triiodothyronine C. an atypical antipsychotic, such as olanzapine (Zyprexa) D. an anticonvulsant, such as gabapentin (Neurontin)

Lithium, triiodothyronine (T3 ), and atypical antipsychotics can all provide clinical improvement when used in conjunction with the ineffective antidepressant. The American Psychiatric Association and the Institute for Clinical Systems Improvement both recommend a trial of *lithium (A)* or low-dose T 3 for patients who have an incomplete response to antidepressant therapy. While thyroid supplementation as adjunctive therapy is effective, the recommended dosage is no higher than 50 μg/day. Atypical antipsychotics can be used as add-on therapy, but are not as effective as lithium or T3. Anticonvulsant medications such as gabapentin have been shown to be effective in the management of bipolar affective disorder, but not as adjunctive therapy in the treatment of unipolar depression resistant to single-agent antidepressants.

A 55-year-old female with diabetes mellitus, hypertension, and hyperlipidemia presents to your office for routine follow-up. Her serum creatinine level is 1.5 mg/dL (estimated creatinine clearance 50 mL/min). Which one of the following diabetes medications would be contraindicated in this patient? (check one) A. Metformin (Glucophage) B. Exenatide (Byetta) C. Acarbose (Precose) D. Insulin glargine (Lantus) E. Pioglitazone (Actos)

Metformin is contraindicated in patients with chronic kidney disease. It should be stopped in females with a creatinine level ≥1.4 mg/dL and in males with a creatinine level ≥1.5 mg/dL. Pioglitazone should not be used in patients with hepatic disease. Acarbose should be avoided in patients with cirrhosis or a creatinine level >2.0 mg/dL. Exenatide is not recommended in patients with a creatinine clearance <30 mL/min. Insulin glargine can be used in patients with renal disease at any stage, but the dosage may need to be decreased.

A mother calls for advice regarding her 2-year-old son. She found an open container of immediate-release diltiazem (Cardizem) on the floor, with some spilled and partially chewed tablets, and estimates that her son opened the container about 90 minutes ago. He does not appear to be in any distress. Which one of the following would you advise her to do? A. Administer syrup of ipecac at home and observe B. Transport the child to the emergency department for gastric lavage C. Transport the child to the emergency department for administration of activated charcoal D. Transport the child to the emergency department for administration of activated charcoal and a cathartic E. Transport the child to the hospital for admission to the pediatric intensive-care unit for observation

More than 9500 cases of calcium channel blocker intoxication were reported to U.S. poison control centers in 2005. Substantial toxicity can occur with one or two tablets, and all children suspected of ingesting a calcium channel blocker should be *admitted to a pediatric intensive-care unit for monitoring and management (E)*. The use of gastric emptying, cathartics, or adsorptive agents is unlikely to be helpful and should be considered only in patients presenting within 1 hour of ingestion, if then. The American Academy of Pediatrics has advised that syrup of ipecac not be kept in the home because of toxicity and dubious benefit.

A 22-year-old female in her second trimester of pregnancy presents with a 48-hour history of a sore throat. She has also had coryza and a nonproductive cough. A physical examination reveals a temperature of 37.3°C (99.2°F) and a blood pressure of 110/70 mm Hg. A HEENT examination reveals tonsillar and pharyngeal erythema with no exudate. There is no adenopathy. Her chest is clear. Which one of the following would be most appropriate for this patient? A. Reassurance and symptomatic treatment only B. A routine throat culture C. A rapid antigen detection test for Streptococcus D. Azithromycin (Zithromax) for 5 days E. Penicillin V for 10 days

Most episodes of pharyngitis are caused by viral rather than bacterial infections. The original Centor score used four signs and symptoms to estimate the probability of acute streptococcal pharyngitis in adults with a sore throat, and was later modified by adding age as a fifth criterion. One point each is assigned for (1) absence of cough, (2) swollen, tender anterior cervical nodes, (3) temperature >38.0°C (100.4°F), and (4) a tonsillar exudate and swelling. One point is added for patients between the ages of 3 and 14 years, and a point is subtracted for patients over the age of 45. The cumulative score determines the likelihood of streptococcal pharyngitis and the need for antibiotics, and guides testing strategies. Patients with a score of zero or 1 are at very low risk for streptococcal pharyngitis and do not require testing or antibiotic therapy. Patients with a score of 2-3 should be tested using a rapid antigen test or throat culture, and a positive result warrants antibiotic therapy. Patients with a score of 4 or higher are at high risk for streptococcal pharyngitis, and empiric treatment may be considered. This patient's score is zero, and *no testing or treatment is warranted (A)*

A 55-year-old male sees you for a follow-up visit for hypercholesterolemia and hypertension. He is in good health, does not smoke, and drinks alcohol infrequently. His medications include a multiple vitamin daily; aspirin, 81 mg daily; lisinopril (Prinivil, Zestril), 10 mg daily; and lovastatin (Mevacor), 20 mg daily. His vital signs are within normal limits except for a BMI of 33.4 kg/m2 . At today's visit his ALT (SGPT) level is 55 IU/L (N 10-45) and his AST (SGOT) level is 44 IU/L (N 10-37). The remainder of the liver panel is normal. Which one of the following is the most likely cause of the elevation in liver enzymes? A. A side effect of lovastatin B. Gallbladder disease C. Hepatitis A D. Alcoholic liver disease E. Metabolic syndrome

Non-alcoholic fatty liver disease (NAFLD) is the most common cause of abnormal liver tests in the developed world. Its prevalence increases with age, body mass index, and triglyceride concentrations, and in patients with diabetes mellitus, hypertension, or insulin resistance. *There is a significant overlap between metabolic syndrome and diabetes mellitus, and NAFLD is regarded as the liver manifestation of insulin resistance.* Statin therapy is considered safe in such individuals and can improve liver enzyme levels and reduce cardiovascular morbidity in patients with mild to moderately abnormal liver tests that are potentially attributable to NAFLD.

Which one of the following treatments for type 2 diabetes mellitus often produces significant weight loss? A. Exenatide (Byetta) B. Glipizide (Glucotrol) C. Pioglitazone (Actos) D. Insulin detemir (Levemir) E. Insulin lispro (Humalog)

Of the many currently available medications to treat diabetes mellitus, only metformin and incretin mimetics such as *exenatide (A)* have the additional benefit of helping the overweight or obese patient lose a significant amount of weight. Most of the other medications, including all the insulin formulations, unfortunately lead to weight gain or have no effect on weight.

A 57-year-old male executive sees you because of "shaky hands." His tremor is most noticeable when he is holding something or writing, and is more prominent in his hand than in his shoulder. He has noticed that it seems better "after a beer or two" at social gatherings. He has no other health problems. On examination you note a very definite tremor when he unbuttons his shirt. His gait is normal and there is no resting tremor. He has no other health problems. On examination you note a very definite tremor when he unbuttons his shirt. His gait is normal and there is no resting tremor. He has a previous history of intolerance to β-blockers. Of the following, which medication would be the best choice for this patient? A. Levodopa/carbidopa (Sinemet) B. Amantadine (Symmetrel) C. Primidone (Mysoline) D. Lithium carbonate

Parkinson's disease and essential tremor are the primary concerns in a person of this age who presents with a new tremor. A coarse, resting, pill-rolling tremor is characteristic of Parkinson's disease. Essential tremor is primarily an action tremor and is a common movement disorder, occurring in members of the same family with a high degree of frequency. Alcohol intake will temporarily cause marked reduction in the tremor. β-Adrenergic blockers have been the mainstay of treatment for these tremors, but this patient is intolerant to these drugs. *Primidone (C)* has been effective in the treatment of essential tremor, and in head-to-head studies with propranolol has been shown to be superior after 1 year.

A 25-year-old male presents to your office with a 1-week history of neck pain with radiation to the left hand, along with intermittent numbness and tingling in the left arm. His history is negative for injury, fever, or lower extremity symptoms. Extension and rotation of the neck to the left while pressing down on the head (Spurling's maneuver) exacerbates the symptoms. His examination is otherwise normal. Cervical radiographs are negative. Which one of the following would be most appropriate at this point? A. NSAIDs for pain relief B. A trial of tricyclic antidepressants C. Cervical corticosteroid injection D. Cervical MRI E. Referral to a spine subspecialist

Patients who present with acute cervical radiculopathy and normal radiographs can be treated conservatively. The vast majority of patients with cervical radiculopathy improve without surgery. Of the interventions listed, *NSAIDs (A)* are the initial treatment of choice. Tricyclic antidepressants, as well as tramadol and venlafaxine, have been shown to help with chronic neuropathic pain. Cervical MRI is not indicated unless there are progressive neurologic defects or red flags such as fever or myelopathy. Likewise, referral to a subspecialist should be reserved for patients who have persistent pain after 6-8 weeks of conservative management and for those with signs of instability. Cervical corticosteroid injections have been found to be helpful in the management of cervical radiculopathy, but should not be administered before MRI is performed.

A 52-year-old Hispanic female with diabetes mellitus and stage 3 chronic kidney disease sees you for follow-up after tests show an estimated glomerular filtration rate of 56 mL/min. Which one of the following medications should she avoid to prevent further deterioration in renal function? (check one) A. Lisinopril (Prinivil, Zestril) B. Folic acid C. Low-dose aspirin D. Candesartan (Atacand) *E. Ibuprofen *

Patients with chronic kidney disease (CKD) and those at risk for CKD because of conditions such as hypertension and diabetes have an increased risk of deterioration in renal function from NSAID use. NSAIDs induce renal injury by acutely reducing renal blood flow and, in some patients, by causing interstitial nephritis. Because many of these drugs are available over the counter, patients often assume they are safe for anyone. Physicians should counsel all patients with CKD, as well as those at increased risk for CKD, to avoid NSAIDs. ACE inhibitors and angiotensin II receptor blockers are renoprotective and their use is recommended in all diabetics. The use of low-dose aspirin and folic acid is recommended in all patients with diabetes, due to the vasculoprotective properties of these drugs. High-dose aspirin should be avoided because it acts as an NSAID.

A 24-year-old primigravida has nausea and vomiting associated with pregnancy. Which one of the following is recommended by the American Congress of Obstetricians and Gynecologists (ACOG) as first-line therapy? A. Droperidol (Inapsine) B. Ondansetron (Zofran) C. Prochlorperazine D. Metoclopramide (Reglan) E. Doxylamine (Unisom) and vitamin B6

Pharmacologic therapies that have been used include vitamin B6 , antihistamines, and prokinetic agents, as well as other medications. Randomized, placebo-controlled trials have shown that vitamin B6 is effective for this problem. The combination of *vitamin B 6 and doxylamine (E)* was studied in more than 6000 patients and was associated with a 70% reduction in nausea and vomiting, with no evidence of teratogenicity. It is recommended by the American Congress of Obstetricians and Gynecologists as first-line therapy for nausea and vomiting in pregnancy. In rare cases, metoclopramide has been associated with tardive dyskinesia, and the FDA has issued a black-box warning concerning the use of this drug in general. The 5-HT3 -receptor antagonists, such as ondansetron, are being used for hyperemesis in pregnancy, but information is limited. Droperidol has been used for this problem in the past, but it is now used infrequently because of its risks, particularly heart arrhythmias.

Which one of the following community health programs best fits the definition of secondary prevention? A. An antismoking education program at a local middle school B. Blood pressure screening at a local church C. A condom distribution program D. Screening diabetic patients for microalbuminuria

Prevention traditionally has been divided into three categories: primary, secondary, and tertiary. Primary prevention targets individuals who may be at risk to develop a medical condition and intervenes to prevent the onset of that condition (e.g., childhood vaccination programs, water fluoridation, antismoking programs, and education about safe sex). Secondary prevention targets individuals who have developed an asymptomatic disease and institutes treatment to prevent complications (e.g., routine Papanicolaou tests; *screening for hypertension (B)*, diabetes, or hyperlipidemia). Tertiary prevention targets individuals with a known disease, with the goal of limiting or preventing future complications (e.g., screening diabetics for microalbuminuria, rigorous treatment of diabetes mellitus, and post-myocardial infarction prophylaxis with β-blockers and aspirin).

A 50-year-old male has a pre-employment chest radiograph showing a pulmonary nodule. There are no previous studies available. Which one of the following would raise the most suspicion that this is a malignant lesion if found on the radiograph? A. The absence of calcification B. Location above the midline of the lung C. A diameter of 4 mm D. A solid appearance

Pulmonary nodules are a common finding on routine studies, including plain chest radiographs, and require evaluation. Radiographic features of benign nodules include a diameter <5 mm, a smooth border, a solid appearance, concentric calcification, and a doubling time of less than 1 month or more than 1 year. Features of malignant nodules include a size >10 mm, an irregular border, a "ground glass" appearance, either *no calcification (A)* or an eccentric calcification, and a doubling time of 1 month to 1 year.

Which one of the following treatments for diabetes mellitus reduces insulin resistance? A. Acarbose (Precose) B. Sitagliptin (Januvia) C. Repaglinide (Prandin) D. Exenatide (Byetta) E. Pioglitazone (Actos)

Repaglinide and nateglinide are nonsulfonylureas that act on a portion of the sulfonylurea receptor to stimulate insulin secretion. *Pioglitazone (E)* is a thiazolidinedione, which reduces insulin resistance. It is believed that the mechanism for this is activation of PPAR-Y, a receptor that affects several insulin-responsive genes. Acarbose is a competitive inhibitor of α-glucosidases, enzymes that break down complex carbohydrates into monosaccharides. This delays the absorption of carbohydrates such as starch, sucrose, and maltose, but does not affect the absorption of glucose. Sitagliptin is a DPP-IV inhibitor, and this class of drugs inhibits the enzyme responsible for the breakdown of the incretins GLP-1 and GIP. Exenatide is an incretin mimetic that stimulates insulin secretion in a glucose-dependent fashion, slows gastric emptying, and may promote satiety.

An 8-year-old male is brought to the emergency department with an acute asthma attack that began 48 hours earlier. His mother initiated his asthma action plan when the attack began, starting oral prednisolone plus albuterol (Proventil, Ventolin) by metered-dose inhaler with a spacer every 3-4 hours. In the emergency department the child is alert, with a respiratory rate of 30 beats/min and an oxygen saturation of 94% on room air. He is audibly wheezing. Peak flow is 40% of the predicted value. A. Continue the current albuterol treatment but switch to a nebulizer B. Administer high-dose albuterol via nebulizer every 20 minutes for 1 hour C. Administer intravenous corticosteroids within the first hour D. Administer magnesium sulfate intravenously E. Prescribe high-dose mucolytics and chest physiotherapy

Repeated doses of a short-acting β2-agonist and correction of hypoxia are the main elements of initial emergency department treatment for acute asthma exacerbations in children. Nebulizer treatments are no better than a metered-dose inhaler with a spacer (SOR A). High-dose nebulized albuterol every 20 minutes for 1 hour has not been shown to be beneficial. In children already receiving standard treatment with albuterol and corticosteroids the addition of *intravenous magnesium sulfate (D)* has been shown to improve lung function and reduce the need for hospitalization (SOR A). Oral administration of corticosteroids is as effective as the intravenous route for reducing the need for hospital admission (SOR A).

A patient with chronic kidney disease presents with chronic normocytic anemia with a hemoglobin level of 7.8 g/dL. The best outcome is predicted if you raise the hemoglobin level to: A. 8-10 g/dL B. 10-12 g/dL C. 12-14 g/dL D. >14 g/dL

Studies have shown that patients who had hemoglobin levels targeted to normal ranges did worse than patients who had hemoglobin levels of *10-12 g/dL (B)*. The incidence of stroke, heart failure, and death increased in patients targeted to normal hemoglobin levels, and there was no demonstrable decrease in cardiovascular events (SOR A).

A 40-year-old businessman has recently been diagnosed with irritable bowel syndrome after extensive testing by his gastroenterologist. His predominant symptoms are diarrhea and pain. Which one of the following has been shown to be helpful in controlled trials? A. Probiotics such as yogurt and buttermilk B. Insoluble fiber such as wheat bran, corn bran, and defatted flaxseed C. Soluble fiber such as psyllium (ispaghula) D. Turmeric E. Peppermint oil

Studies suggest that in 25% of patients, irritable bowel syndrome may be caused or aggravated by one or more dietary components. Despite its popularity, fiber is marginally beneficial and insoluble fiber may worsen symptoms in patients with diarrhea. Probiotics in the form of foods such as buttermilk and live-culture yogurt have thus far not been established as useful. Daily use of *peppermint oil (E)* has been shown to relieve symptoms.

Which one of the following would suggest that the sudden and unexpected death of a healthy infant resulted from deliberate suffocation rather than sudden infant death syndrome? A. No previous history of apneic episodes B. An age of 9 months C. Mottled skin D. Clenched fists E. Blood-tinged froth in the mouth

Sudden infant death syndrome (SIDS) is the most common cause of death during the first 6 months of life in the United States, with a peak incidence at 2-4 months of age and a quick dropoff by the age of 6 months. The cause of death is a retrospective diagnosis of exclusion, and is supported by a history of quiet death during sleep in a previously healthy infant younger than 6 months of age. Evidence of terminal activity may be present, such as clenched fists or a serosanguineous, blood-tinged, or mucoid discharge from the mouth or nose. Lividity and mottling are frequently present in dependent areas. The reported history and autopsy findings of deliberate suffocation may mirror the findings of SIDS, but suffocation should be considered when there is documentation of any of the following: *infant age older than 6 months (B)*, previous similar sibling deaths, simultaneous twin deaths, or evidence of pulmonary hemorrhage. A history of recurrent apnea or cyanosis has not been causally linked to SIDS; when such reported events have only been witnessed by one caretaker, deliberate suffocation should be suspected.

Which one of the following is diagnostic for type 2 diabetes mellitus? *A. A fasting plasma glucose level ≥126 mg/dL on two separate occasions* B. An oral glucose tolerance test (75-g load) with a 2-hour glucose level ≥160 mg/dL C. A random blood glucose level ≥200 mg/dL on two occasions in an asymptomatic person D. A hemoglobin A 1c ≥6.0% on two separate occasions

The ADA recommends screening for all asymptomatic adults with a BMI >25.0 kg/m whohave one or more additional risk factors for diabetes mellitus, and screening for all adults with no risk factors every 3 years beginning at age 45. Current criteria for the diagnosis of diabetes mellitus include a hemoglobin A1c≥6.5%, a fasting plasma glucose level ≥126 mg/dL, a 2-hour plasma glucose leve l≥200 mg/dL, or, in a symptomatic patient, a random blood glucose level ³200 mg/dL. In the absence of unequivocal hyperglycemia, results require confirmation by repeat testing.

A 26-year-old female calls your office to inquire about the results of her recent Papanicolaou (Pap) test. The report indicates the presence of atypical squamous cells of undetermined significance (ASC-US), and her reflex HPV test is negative for high-risk HPV types. The patient has never had an abnormal Pap test and has had three normal tests over the past 6 years. She is a nonsmoker. You advise the patient that the most appropriate next step would be to: A. repeat the Pap test every 3 months for 1 year B. repeat the Pap test in 6 months and 12 months *C. repeat the Pap test in 12 months* D. continue routine Pap tests, with the next test in 3 years E. schedule colposcopy as soon as possible

The ASC-US/LSIL Triage Study (ALTS) demonstrated that there are three appropriate follow-up options for managing women with an ASC-US Papanicolaou (Pap) test result: (1) two repeat cytologic examinations performed at 6-month intervals; (2) reflex testing for HPV; or (3) a single colposcopic examination. This expert consensus recommendation has been confirmed in more recent clinical studies, additional analyses of the ALTS data, and meta-analyses of published studies (SOR A). Reflex HPV testing refers to testing either the original liquid-based cytology residual specimen or a separate sample collected for HPV testing at the time of the initial screening visit. This approach eliminates the need for women to return to the office or clinic for repeat testing, rapidly reassures women who do not have a significant lesion, spares 40%-60% of women from undergoing colposcopy, and has been shown to have a favorable cost-effectiveness ratio. In this patient's case, the HPV testing was negative, and there is no need to repeat the Pap test at 6-month intervals or to perform colposcopy.

A patient is sent to you by his employer after falling down some steps and twisting his ankle and foot. Which one of the following would be the most appropriate reason to obtain foot or ankle radiographs? A. Notable swelling and discoloration over the anterior talofibular ligament B. A complaint of marked pain with weight bearing as he walks into the examining room *C. Pain in the maleolar zone and bone tenderness of the posterior medial malleolus* D. The absence of passive plantar foot flexion when the calf is squeezed (Thompson test)

The Ottawa ankle and foot rules are prospectively validated decision rules that help clinicians decrease the use of radiographs for foot and ankle injuries without increasing the rate of missed fracture. The rules apply in the case of blunt trauma. According to these guidelines, an ankle radiograph series is required only if there is pain in the malleolar zone and bone tenderness of either the distal 6 cm of the posterior edge or the tip of either the lateral malleolus or the medial malleolus. Inability to bear weight for four steps, both immediately after the injury and in the emergency department, is also an indication for ankle radiographs. Foot radiographs are required only if there is pain in the midfoot zone and bone tenderness at the base of the 5th metatarsal or the navicular, or if the patient is unable to bear weight both immediately after the injury and in the emergency department.

A 21-year-old sexually active female presents with acute pelvic pain of several days' duration. A pelvic examination reveals right-sided tenderness and a general fullness in that area. In addition to laboratory testing, you decide to order an imaging study. Which one of the following is the best choice at this time? A. Transabdominal ultrasonography B. Transvaginal ultrasonography C. Contrast CT of the abdomen and pelvis D. Hysteroscopy E. Hysterosalpingography

The best initial imaging study for acute pelvic pain in women is *transvaginal ultrasonography (B)*. This provides the greatest level of detail regarding the uterus and adnexae, superior to transcutaneous ultrasonography. CT of the abdomen/pelvis and hysterosalpingography may be indicated eventually in some patients with pelvic pain, but they are not the initial studies of choice. Hysteroscopy is not routinely used in the evaluation of pelvic pain.

The FDA has imposed a black box warning on all thiazolidinediones, such as pioglitazone (Actos). This warning addresses a contraindication to the prescription of these drugs in patients with: A. renal insufficiency B. dementia C. exposure to radiocontrast media D. heart failure E. respiratory failure

The black box warning for thiazolidinediones specifically addresses *heart failure (D)*. These agents are also contraindicated in patients with type 1 diabetes mellitus or hepatic disease, and in premenopausal anovulatory women.

The parents of a young child ask your advice about the need for fluoride supplementation in order to prevent tooth decay. Which one of the following is true regarding current U.S. Preventive Services Task Force guidelines for fluoride supplementation? A. It is not recommended due to potential fluoride toxicity B. Dental fluoride varnish is too toxic for routine use C. Oral fluoride supplementation is recommended if the primary drinking water source is low in fluoride D. Fluoridated toothpaste provides adequate protection if used as soon as the child has teeth E. The need for fluoride supplementation is determined by serum fluoride levels

The current (2004) recommendation of the U.S. Preventive Services Task Force (USPSTF) is that children over the age of 6 months receive oral fluoride supplementation if the primary drinking water source is deficient in fluoride. The USPSTF cites "fair" evidence (B recommendation) that such supplementation reduces the incidence of dental caries and concludes that the overall benefit outweighs the potential harm from dental fluorosis. Dental fluorosis is chiefly a cosmetic staining of the teeth, is uncommon with currently recommended fluoride intake, and has no other functional or physiologic consequences. Fluoridated toothpaste can cause fluorosis in children younger than 2 years of age, and is therefore not recommended in this age group. Fluoridated toothpaste by itself does not reliably prevent tooth decay. Fluoride varnish, applied by a dental or medical professional, is another treatment option to prevent caries. It provides longer-lasting protection than fluoride rinses, but since it is less concentrated, it may carry a lower risk of fluorosis than other forms of supplementation.

A 24-year-old male presents with a fever of 38.9°C (102.0°F), generalized body aches, a sore throat, and a cough. His symptoms started 24 hours ago. He is otherwise healthy. You suspect novel influenza A H1N1 infection, as there have been numerous cases in your community recently. A rapid influenza diagnostic test is positive, and you recommend over-the-counter symptomatic treatment. You see him 2 days later after he is admitted to the hospital through the emergency department with dehydration and mild respiratory distress. A specimen is sent to the state laboratory for PCR testing. Which one of the following would be most appropriate at this point? (check one) A. Oseltamivir (Tamiflu) B. Zanamivir (Relenza) C. Amantadine (Symmetrel) D. Rimantadine (Flumadine) E. No antiviral treatment

The currently circulating novel influenza A H1N1 virus is almost always susceptible to neuraminidase inhibitors (oseltamivir and zanamivir) and resistant to the adamantanes (amantadine and rimantadine). Zanamivir should not be used in patients with COPD, asthma, or respiratory distress. *Antiviral treatment of influenza is recommended for all persons with clinical deterioration requiring hospitalization, even if the illness started more than 48 hours before admission*. Antiviral treatment should be started as soon as possible. Waiting for laboratory confirmation is not recommended.

A patient presents with a pigmented skin lesion that could be a melanoma. Its largest dimension is 0.5 cm. What should be the first step in management? A. A shave biopsy B. Excision with a 1-mm margin C. Wide excision with a 1-cm margin D. Wide excision with a 1-cm margin E. Excision with sentinel node dissection

The diagnosis of melanoma should be made by *simple excision with clear margins (B)*. A shave biopsy should be avoided because determining the thickness of the lesion is critical for staging. Wide excision with or without node dissection is indicated for confirmed melanoma, depending on the findings from the initial excisional biopsy.

One of your patients has been diagnosed with monoclonal gammopathy of undetermined significance (MGUS). Which one of the following is used to determine whether his condition has progressed to multiple myeloma? A. The length of time since the diagnosis of MGUS was made B. The level of M protein C. The percentage of plasma cells in bone marrow D. Evidence of end-organ damage

The diagnosis of multiple myeloma is based on evidence of myeloma-related end-organ impairment in the presence of M protein, monoclonal plasma cells, or both. This evidence may include hypercalcemia, renal failure, anemia, or skeletal lesions. Monoclonal gammopathy of undetermined significance does not progress steadily to multiple myeloma. There is a stable 1% annual risk of progression.

A 17-year-old female sees you for a preparticipation evaluation. She has run 5 miles a day for the last 6 months, and has lost 6 lb over the past 2 months. Her last menstrual period was 3 months ago. Other than the fact that she appears to be slightly underweight, her examination is normal. To fit the criteria for the female athlete triad, she must have which one of the following? A. A formal diagnosis of an eating disorder B. Amenorrhea for 1 year C. A Z-score on bone-density testing of -2.5 or less D. Withdrawal bleeding after progesterone administration E. A history of a stress fracture resulting from minimal trauma

The initial definition of the female athlete triad was amenorrhea, osteoporosis, and disordered eating. The American College of Sports Medicine modified this in 2007, emphasizing that the triad components occur on a continuum rather than as individual pathologic conditions. The definitions have therefore expanded. Disordered eating is no longer defined as the formal diagnosis of an eating disorder. Energy availability,defined as dietary energy intake minus exercise energy expenditures, is now considered a risk factor for the triad, as dietary restrictions and substantial energy expenditures disrupt pituitary and ovarian function. Primary amenorrhea is defined as lack of menstruation by age 15 in females with secondary sex characteristics. Secondary amenorrhea is the absence of three or more menstrual cycles in a young woman previously experiencing menses. For those with secondary amenorrhea, a pregnancy test should be performed. If this is not conclusive, a progesterone challenge test may be performed. If there is withdrawal bleeding, the cause would be anovulation. Those who do not experience withdrawal bleeding have hypothalamic amenorrhea, and fit one criterion for the triad. Athletes who have amenorrhea for 6 months, disordered eating, and/or a *history of a stress fracture resulting from minimal trauma (E)* should have a bone density test. Low bone mineral density for age is the term used to describe at-risk female athletes with a Z-score of -1 to -2. Osteoporosis is defined as having clinical risk factors for experiencing a fracture, along with a Z-score <-2.

A 12-year-old male middle-school wrestler comes to your office complaining of a recurrent painful rash on his arm. There appear to be several dry vesicles. The most likely diagnosis is which one of the following? A. Molluscum contagiosum B. Human papillomavirus C. Herpes gladiatorum D. Tinea corporis E. Mat burn

The most common infection transmitted person-to-person in wrestlers is *herpes gladiatorum (C)* caused by the herpes simplex virus. Molluscum contagiosum causes keratinized plugs. Human papillomavirus causes warts. Tinea corporis is ringworm, which is manifested by round to oval raised areas with central clearing. Mat burn is an abrasion.

A 52-year-old male with stable coronary artery disease and controlled hypertension sees you for a routine visit and asks for advice regarding prevention of altitude illness for his upcoming trip to Bhutan to celebrate his anniversary. His medical chart indicates that he had a reaction to a sulfa drug in the past. Which one of the following would be most appropriate? A. Advise the patient to not make the trip B. Recommend ginkgo biloba C. Prescribe acetazolamide D. Prescribe dexamethasone

The most common manifestation is acute mountain sickness, heralded by malaise and headache. Risk factors include young age, residence at a low altitude, rapid ascent, strenuous physical exertion, and a previous history of altitude illness. However, activity restriction is not necessary for patients with coronary artery disease who are traveling to high altitudes (SOR C). Ginkgo bilboa is not recommended. Acetazolamide is an effective prophylactic agent (SOR B), but is contraindicated in patients with a sulfa allergy. If used, it should be started a minimum of one day before ascent and continued until the patient acclimatizes at the highest planned elevation. *Dexamethasone (D)* is an effective prophylactic and treatment agent (SOR B), and it is not contraindicated for those with a sulfa allergy. It would be the best option for this patient.

Which one of the following antihypertensive drugs is most likely to cause ankle edema? A. Hydrochlorothiazide B. Amlodopine (Norvasc) C. Lisinopril (Prinivil, Zestril) D. Losartan (Cozar) E. Atenolol (Tenormin)

The most common side effects of calcium channel blockers, such as *amlodipine (B)*, are due to vasodilation. One result of this may be peripheral edema, but it can also cause dizziness, nausea, hypotension, cough, and pulmonary edema. These problems may decrease with time, with reductions in dosage, or with the addition of a diuretic or second calcium antagonist. Other classes of drugs are not associated with these problems.

A 36-year-old member of the National Guard who has just returned from Iraq consults you because of several "boils" on the back of his neck that have failed to heal over the last 6 months, despite two week-long courses of cephalexin (Keflex). You observe three 1- to 2-cm raised minimally tender lesions with central ulceration and crust formation. He denies any fever or systemic symptoms. The most likely cause of these lesions is: A. Pyogenic granuloma B. Leishmaniasis C. Atypical mycobacterial infection D. Squamous cell carcinoma E. Epidermal inclusion cysts

The most likely diagnosis is cutaneous *leishmaniasis (B)*, caused by an intracellular parasite transmitted by the bite of small sandflies. Lesions develop gradually, and are often misdiagnosed as folliculitis or as infected epidermal inclusion cysts, but they fail to respond to usual skin antibiotics. Hundreds of cases have been diagnosed in troops returning from Iraq, most due to Leishmania major. Treatment is not always required, as most lesions will resolve over several months; however, scarring is frequent. U.S. military medical facilities and the CDC are coordinating treatment when indicated with sodium stibogluconate.

A 55-year-old white male comes to your office with weakness and a headache. He also describes an annoying pruritus that occurs frequently after he takes a hot shower. The physical examination is remarkable for the presence of an enlarged spleen. He has a hemoglobin level of 21 g/dL (N 12-16) and a hematocrit of 63% (N 36-48). To confirm your clinical diagnosis, you obtain additional studies. Which one of the following would be most consistent with the most likely diagnosis in this patient? A. A low serum erythropoietin level B. A low platelet count C. A low arterial oxygen concentration D. An elevated carboxyhemoglobin level

The patient described in this case has polycythemia vera. Pruritus after a hot shower (aquagenic pruritus) and the presence of splenomegaly helps to clinically distinguish polycythemia vera from other causes of erythrocytosis (hematocrit >55%). Specific criteria for the diagnosis of polycythemia vera include an elevated red cell mass, a normal arterial oxygen saturation (>92%), and the presence of splenomegaly. In addition, patients usually exhibit thrombocytosis (platelet count >400,000/mm3 ), leukocytosis (WBC>12,000/mm3 ), a *low serum erythropoietin level (A)*, and an elevated leukocyte alkaline phosphatase score. High carboxyhemoglobin levels are associated with secondary polycythemia.

The best drug treatment for symptomatic mitral valve prolapse is: A. quinidine *B. propranolol (Inderal)* C. digoxin D. procainamide E. phenytoin (Dilantin)

The primary treatment for symptomatic mitral valve prolapse is β-blockers. Quinidine and digoxin were used to treat this problem in the past, especially if sinus bradycardia or cardiac arrest occurred with administration of propranolol. Procainamide and phenytoin have not been used to treat this syndrome. Asymptomatic patients require only routine monitoring, while those with significant mitral regurgitation may require surgery. Some patients with palpitations can be managed with lifestyle changes such as elimination of caffeine and alcohol. Orthostatic hypotension can often be managed with volume expansion, such as by increasing salt intake.

A 23-month-old child is brought to your office with a 2-day history of a fever to 102°F (39°C), cough, wheezing, and mildly labored breathing. He has no prior history of similar episodes and there is no improvement with administration of an aerosolized bronchodilator. Which one of the following is now indicated? A. Bronchodilator aerosol treatment every 6 hours B. Corticosteroids C. An antibiotic D. A decongestant E. Supportive care only

This child has typical findings of bronchiolitis. The initial infection usually occurs by the age of 2 years. It is caused by respiratory syncytial virus (RSV). Bronchodilator treatment may be tried once and discontinued if there is no improvement. Treatment usually consists of *supportive care only (E)*, including oxygen and intravenous fluids if indicated. RSV infection may recur, since an infection does not provide immunity. Up to 10% of infected children will have wheezing past age 5, and bronchiolitis may predispose them to asthma.

A 3-day-old female developed a rash 1 day ago that has continued to progress and spread. The infant was born at term after an uncomplicated pregnancy and delivery to a healthy mother following excellent prenatal care. The infant was discharged 2 days ago in good health. She does not appear to be irritable or in distress, and she is afebrile and feeding well. On examination, abnormal findings are confined to the skin, including her face, trunk, and proximal extremities, which have macules, papules, and pustules that are all 2-3 mm in diameter. Her palms and soles are spared. A stain of a pustular smear shows numerous eosinophils. Which one of the following is the most likely diagnosis? A. Staphylococcal pyoderma B. Herpes simplex C. Acne neonatorum D. Erythema toxicum neonatorum E. Rocky Mountain spotted fever

This infant has a typical presentation of *erythema toxicum neonatorum (D)*. Staphylococcal pyoderma is vesicular and the stain of the vesicle content shows polymorphonuclear leukocytes and clusters of gram-positive bacteria. Because the mother is healthy and the infant shows no evidence of being otherwise ill, systemic infections such as herpes are unlikely. Acne neonatorum consists of closed comedones on the forehead, nose, and cheeks. Rocky Mountain spotted fever is a tickborne disease that does not need to be considered in a child who is not at risk.

You examine an 11-month-old male who has had several paroxysms of abdominal pain in the last 2 hours. The episodes last 1-2 minutes; the infant screams, turns pale, and doubles up. Afterward, he seems normal. A physical examination is normal except for a possible fullness in the right upper quadrant of the abdomen. The most likely diagnosis is: (check one) A. pyloric stenosis B. choledochal cyst C. Meckel's diverticulum D. intussusception E. intestinal malrotation

This is a classic presentation for *intussusception (D)*, which usually occurs in children under the age of 2 years and is characterized by paroxysms of colicky abdominal pain. A mass is palpable in about two-thirds of patients. Pyloric stenosis presents with a palpable mass, but usually develops between 4 and 6 weeks of age. A choledochal cyst presents with the classic triad of right upper quadrant pain, jaundice, and a palpable mass. Meckel's diverticulum usually presents in this age group with painless lower gastrointestinal bleeding. Intestinal malrotation usually presents within the first 4 weeks of life and is characterized by bilious vomiting.

A 36-year-old male complains of clear rhinorrhea, nasal congestion, and watery, itchy eyes for several months. Tests in the past have suggested that he has an allergy to dust mites. Which one of the following is most likely to provide the most relief from his symptoms? A. Oral antihistamines B. An oral leukotriene-receptor antagonist C. Intranasal antihistamines D. Intranasal corticosteroids E. Furnace filters and mite-proof bedding covers

This patient has classic symptoms of allergic rhinitis. *Intranasal corticosteroids (D)* are considered the mainstay of treatment for mild to moderate cases. In multiple studies, intranasal corticosteroid sprays have proven to be more efficacious than the other options listed, even for ocular symptoms. Air filtration systems and bedding covers have not been shown to reduce symptoms.

A 5-year-old white male has an itchy lesion on his right foot. He often plays barefoot in a city park that is subject to frequent flooding. The lesion is located dorsally between the web of his right third and fourth toes, and extends toward the ankle. It measures approximately 3 cm in length, is erythematous, and has a serpiginous track. The remainder of his examination is within normal limits. Which one of the following is the most likely cause of these findings? A. Dog or cat hookworm (Ancylostoma species) B. Dog or other canid tapeworm (Echinococcus granulosus) C. Cat protozoa (Toxoplasma gondii) D. Dog or cat roundworm (Toxocara canis or T. mystax)

This patient has cutaneous larva migrans, a common condition caused by *dog and cat hookworms (A)*. Fecal matter deposited on soil or sand may contain hookworm eggs that hatch and release larvae, which are infective if they penetrate the skin. Walking barefoot on contaminated ground can lead to infection. Echinococcosis (hydatid disease) is caused by the cestodes (tapeworms) Echinococcus granulosus and Echinococcus multilocularis, found in dogs and other canids. It infects humans who ingest eggs that are shed in the animals feces and results in slow-growing cysts in the liver or lungs, and occasionally in the brain, bones, or heart. Toxoplasmosis is caused by the protozoa Toxoplasma gondii, found in cat feces. Humans can contract it from litter boxes or feces-contaminated soil, or by consuming infected undercooked meat. It can be asymptomatic, or it may cause cervical lymphadenopathy, a mononucleosis-like illness; it can also lead to a serious congenital infection if the mother is infected during pregnancy, especially during the first trimester. Toxocariasis due to Toxocara canis and Toxocara cati causes visceral or ocular larva migrans in children who ingest soil contaminated with animal feces that contains parasite eggs, often found in areas such as playgrounds and sandboxes.

A 44-year-old female who suffers from obstructive sleep apnea complains of gradual swelling in her legs over the last several weeks. Her vital signs include a BMI of 44.1 kg/m2 , a respiratory rate of 12/min, a blood pressure of 120/78 mm Hg, and an O 2 saturation of 86% on room air. An EKG and a chest radiograph are normal. Pulmonary function testing shows a restrictive pattern with no signs of abnormal diffusion. Abnormal blood tests include only a significantly elevated bicarbonate level. Which one of the following treatments is most likely to reduce this patient's mortality rate? A. ACE inhibitors B. Routine use of nebulized albuterol (AccuNeb) C. High-dose diuretic therapy D. Continuous oxygen therapy E. Continuous or bilevel positive airway pressure (CPAP or Bi-PAP)

This patient has obesity-hypoventilation syndrome, often referred to as Pickwickian syndrome. These patients are obese (BMI >30 kg/m 2 ), have sleep apnea, and suffer from chronic daytime hypoxia andcarbon dioxide retention. They are at increased risk for significant respiratory failure and death compared to patients with otherwise similar demographics. Treatment consists of nighttime positive airway pressure in the form of *continuous (CPAP) or bi-level (BiPAP) devices, (E)* as indicated by sleep testing. The more hours per day that patients can use this therapy, the less carbon dioxide retention and less daytime hypoxia will ensue. Several small studies suggest that the increased mortality risk from obesity-hypoventilation syndrome can be decreased by adhering to this therapy. The use of daytime oxygen can improve oxygenation, but is not considered adequate to restore the chronic low respiratory drive that is characteristic of this condition.

A 7-year-old female with a history of asthma is brought to your office for a routine follow-up visit. She has a history of exercise-induced asthma, but also has had exacerbations in the past that were unrelated to exercise. In the past month, she has premedicated herself with albuterol (Proventil, Ventolin) with a spacer before recess 5 days/week as usual. She has also needed her albuterol to treat symptoms (wheezing and/or shortness of breath) once or twice per week and had one exacerbation requiring medical treatment in the past year. She has had no nighttime symptoms. Albuterol as needed is her only medication. After reinforcing asthma education, which one of the following would be most appropriate? A. Referral to an asthma specialist B. Addition of a low-dose inhaled corticosteroid C. Addition of a long-acting β-agonist D. Elimination of premedication with albuterol, restricting use to an as-needed basis E. No changes to her regimen

This patient's asthma is well-controlled according to the 2007 NHLBI asthma guidelines. The "rule of twos" is useful in assessing asthma control: in children under the age of 12, asthma is NOT well-controlled if they have had symptoms or used a β-agonist for symptom relief more than twice per week, had two or more nocturnal awakenings due to asthma symptoms in the past month, or had two or more exacerbations requiring systemic corticosteroids in the past year. For individuals over 12 years of age, there must be more than two nocturnal awakenings per month to classify their asthma as not well controlled. Exercise-induced asthma is considered separately. A β-agonist used as premedication before exercise is not a factor when assessing asthma control. Since this patient does not exceed the rule of twos, her asthma is categorized as well-controlled and *no changes to her therapy are indicated (E)*. Asthma education should be reinforced at every visit.

A 75-year-old male consults you after his family expresses concern about his loss of interest in his usual activities. They believe he has become increasingly withdrawn since the death of his wife 8 months earlier. You note he has lost 8 kg (18 lb) since his last office visit 6 months earlier. He does not drink alcohol. His physical examination is unremarkable for his age except for a blood pressure of 105/70 mm Hg. Detailed laboratory studies, including thyroid function tests, are all within normal limits. He tells you he would be fine if he could just get some sleep. His Mini-Mental State Examination is normal, but he is obviously clinically depressed. The most appropriate medication for his depression would be: A. trazodone (Oleptro) B. mirtazapine (Remeron) C. bupropion (Wellbutrin) D. amitriptyline E. nortriptyline (Pamelor)

Trazodone may be useful for insomnia, but is not recommended as a primary antidepressant because it causes sedation and orthostatic hypotension at therapeutic doses. Bupropion would aggravate this patient's insomnia. Tricyclic antidepressants may be effective, but are no longer considered first-line treatments because of side effects and because they can be cardiotoxic. *Mirtazapine (B)* has serotonergic and noradrenergic properties and is associated with increased appetite and weight gain. It may be particularly useful for patients with insomnia and weight loss.

Two doses of varicella vaccine are recommended for: A. adults under 60 years of age who develop shingles B. all children with normal immune status C. only immunocompromised individuals D. only children between 12 months and 13 years of age

Two doses of varicella vaccine are recommended for *all children unless they are immunocompromised (B)*, in which case they should not be immunized against varicella, or with other live-virus vaccines.

A 21-year-old female complains of bulging veins in her right shoulder region, along with swelling and a "tingling" sensation in her right arm that has developed over the past 2 days. There were no unusual events other than her regular workouts with her swim team. Ultrasonography confirms an upper extremity deep-vein thrombosis of her right axillary vein. Which one of the following would be the most appropriate treatment? A. Intravenous heparin for 72 hours, followed by oral warfarin (Coumadin) for 3 months B. Low molecular weight heparin (LMWH) subcutaneously for 5 days only C. LMWH subcutaneously for at least 5 days, followed by oral warfarin for 3 months D. LMWH subcutaneously for at least 5 days, followed by oral warfarin indefinitely E. Oral warfarin for 3 months

Upper extremity deep-vein thrombosis (UE-DVT) accounts for 4% of all cases of DVT. Catheter-related thromboses make up the majority of these cases. Occult cancer, use of oral contraceptives, and inheritable thrombophilia are other common explanations. Another proposed risk factor is the repetitive compression of the axillary-subclavian vein in athletes or laborers, which is the most likely cause of this patient's UE-DVT. Taken as a whole, UE-DVT is generally associated with fewer venous complications, including less chance for thromboembolism, postphlebitic syndrome, and recurrence compared to lower-extremity deep-vein thrombosis (LE-DVT). However, the rates of these complications are still high enough that most experts recommend treatment identical to that of LE-DVT. Specifically, *heparin should be given for 5 days, and an oral vitamin-K antagonist for at least 3 months (C)*.

A 4-year-old is brought to the emergency department with abdominal pain and is noted to have 3+ proteinuria on a dipstick. Three days later the pain has resolved spontaneously, and a repeat urinalysis in your office shows 2+ proteinuria with normal findings on microscopic examination. A metabolic panel, including creatinine and total protein, is also normal. Which one of the following would be most appropriate at this point? A. Renal ultrasonography B. A spot first morning urine protein/creatinine ratio C. An antinuclear antibody and complement panel D. Referral to a nephrologist

When proteinuria is noted on a dipstick and the history, examination, full urinalysis, and serum studies suggest no obvious underlying problem or renal insufficiency, a *urine protein/creatinine ratio (B)* is recommended. This test correlates well with 24-hour urine protein, which is particularly difficult to collect in a younger patient. Renal ultrasonography is appropriate once renal insufficiency or nephritis is established. If pathogenic proteinuria is confirmed, an antinuclear antibody and/or complement panel may be indicated. A nephrology referral is not necessary until the presence of kidney disease or proteinuria from a cause other than benign postural proteinuria is confirmed.

A 20-year-old white female presents with painful and frequent urination that has had a gradual onset over the past week. She has never had a urinary tract infection. There is no associated hematuria, flank pain, suprapubic pain, or fever. She says she has not noted any itching or vaginal discharge. A midstream urine specimen taken earlier in the week showed significant pyuria but a culture was reported as no growth. She has taken an antibiotic for 2 days without relief. Her only other medication is an oral contraceptive agent. Which one of the following is the most likely infectious agent? A. Escherichia coli B. Chlamydia trachomatis C. Candida albicans D. Staphylococcus saprophyticus

Women who present with symptoms of acute dysuria, frequency, and pyuria do not always have bacterial cystitis. In fact, up to 30% will show either no growth or insignificant bacterial growth on a midstream urine culture. Most commonly these patients represent cases of sexually transmitted urethritis caused by *Chlamydia trachomatis (B)*, Neisseria gonorrhoeae, or herpes simplex virus. In this case, the gradual onset, absence of hematuria, and week-long duration of symptoms suggest a sexually transmitted disease. A history of a new sexual partner or a finding of mucopurulent cervicitis would confirm the diagnosis. Empiric treatment with a tetracycline and a search for other sexually transmitted diseases would then be indicated. Another possible diagnosis is urinary tract infection with Escherichia coli or Staphylococcus species; however, the onset of these infections is usually abrupt and accompanied by other signs, such as suprapubic pain or hematuria. Candida is unlikely because there is no accompanying discharge or itching, and the patient's symptoms predate the use of antibiotics.


संबंधित स्टडी सेट्स

MGT Chap 13, MGT Chap 15, MGT Chapter 16, Chapt 12 Mgt, MGT Chap 14

View Set

Exercise Physiology - Introduction Chapter 00

View Set

S-IV Sentence Pattern (Present tense)

View Set

Ch. 5 Quiz "Sociology In Our Times"

View Set

NVNU. Antipodagriniai ir antireumatiniai vaistai.

View Set

abdominal aortic aneurysm practice questions

View Set

History Social Impact of the Industrial revolution

View Set